How can I prove the inequality for integrals of integrable vectors?

Click For Summary
The discussion focuses on proving the inequality | ∫ f dμ | ≤ ∫ | f | dμ for integrable vector functions. The user attempts to use the triangle inequality but struggles to finalize the proof, particularly in relating the L^1 norm to the L^2 norm. They derive an expression involving the sum of integrals of the components but cannot complete the transition to the desired form. The conversation highlights the applicability of various inequalities, including those for L^p norms, in establishing the proof. Assistance is requested to clarify the final steps needed to complete the proof for the L^2 norm.
dataphile
Messages
2
Reaction score
0

Homework Statement


Let f: X \rightarrow \mathbb{R}^{n}. We say that \textbf{f} = (f_{1}, ..., f_{n}) is integrable if each component f_{k} is integrable, and define \int \textbf{f} d\mu = (\int f_{1} d\mu, ..., \int f_{n} d\mu). Show that if \textbf{f} is integrable, then | \int \textbf{f} d\mu | \leq \int | \textbf{f} | d\mu.


Homework Equations


The chapter discusses various inequalities for integrals, including Jensen's inequality, Holder's inequality, the definition of an L^{p} norm, Minkowski's inequality, etc.


The Attempt at a Solution


The inequality is saying that the magnitude of the integral of a vector is less than or equal to taking the integral of the magnitude of the vector. To do the formal proof, I've experimented with using the triangle inequality, but can't seem to get everything in the right place. The closest I have is this:

| \int \textbf{f} d\mu | \leq \sum_{i=1}^{n} | \int f_{i} d\mu | (by the triangle inequality)

\sum_{i=1}^{n} | \int f_{i} d\mu | \leq \sum_{i=1}^{n} \int |f_{i}| d\mu = \int \sum_{i=1}^{n} |f_{i}| d\mu

So I end up with something in the L^1 norm and can't make the last steps to get to \int | \textbf{f} | d\mu = \int ((\sum_{i=1}^{n} f_{i}^2)^\frac{1}{2}) d\mu

I would be grateful if someone could just point me in the right direction.
 
Physics news on Phys.org
All your inequalities are valid for all Lp norms, 1,2, ... \inf
 
I know that the inequality will hold true for any L^p norm, but I am still having trouble finishing the proof for the particular case of the L^2 norm.
 
Question: A clock's minute hand has length 4 and its hour hand has length 3. What is the distance between the tips at the moment when it is increasing most rapidly?(Putnam Exam Question) Answer: Making assumption that both the hands moves at constant angular velocities, the answer is ## \sqrt{7} .## But don't you think this assumption is somewhat doubtful and wrong?

Similar threads

  • · Replies 7 ·
Replies
7
Views
2K
  • · Replies 3 ·
Replies
3
Views
2K
  • · Replies 105 ·
4
Replies
105
Views
6K
  • · Replies 1 ·
Replies
1
Views
2K
Replies
10
Views
2K
  • · Replies 4 ·
Replies
4
Views
4K
  • · Replies 6 ·
Replies
6
Views
3K
  • · Replies 5 ·
Replies
5
Views
2K
  • · Replies 3 ·
Replies
3
Views
2K
  • · Replies 14 ·
Replies
14
Views
2K